master > master: Bemerkgung in Quiz9 überarbeitet

This commit is contained in:
RD 2021-01-20 11:14:24 +01:00
parent a526a4b106
commit b034eefab5
2 changed files with 49 additions and 46 deletions

Binary file not shown.

View File

@ -2715,8 +2715,7 @@ und daraus die Parameter abzulesen.
1&-2&4&0\\
0&11&-15&1\\
0&0&-7&1\\
\end{smatrix}
\\
\end{smatrix}\\
\end{mathe}
Wende die Zeilentransformation
@ -2728,8 +2727,7 @@ und daraus die Parameter abzulesen.
1&-2&4&0\\
0&11&-8&0\\
0&0&-7&1\\
\end{smatrix}
\\
\end{smatrix}\\
\end{mathe}
Aus der Zeilenstufenform erschließt sich, dass $t_{4}$ frei ist.
@ -7523,22 +7521,22 @@ Für jeden Fall berechnen wir $\ggT(a,b)$ mittels des Euklidischen Algorithmus
$a$ &$b$ &Restberechnung (symbolisch) &Restberechnung (Werte)\\
\hline
\endhead
$1529$ &$170$ &$a = b\cdot q_{1} + r_{1}$ &$1529 = 170\cdot 8 + 169$\\
&&$b = r_{1}\cdot q_{2} + r_{2}$ &$170 = 169\cdot 1 + \boxed{\mathbf{1}}$\\
&&$r_{1} = r_{2}\cdot q_{3} + r_{3}$ &$169 = 1\cdot 169 + 0$\\
\hline
$13758$ &$21$ &$a = b\cdot q_{1} + r_{1}$ &$13758 = 21\cdot 655 + \boxed{\mathbf{3}}$\\
&&$b = r_{1}\cdot q_{2} + r_{2}$ &$21 = 3\cdot 7 + 0$\\
\hline
$210$ &$45$ &$a = b\cdot q_{1} + r_{1}$ &$210 = 45\cdot 4 + 30$\\
&&$b = r_{1}\cdot q_{2} + r_{2}$ &$45 = 30\cdot 1 + \boxed{\mathbf{15}}$\\
&&$r_{1} = r_{2}\cdot q_{3} + r_{3}$ &$30 = 15\cdot 2 + 0$\\
\hline
$1209$ &$102$ &$a = b\cdot q_{1} + r_{1}$ &$1209 = 102\cdot 11 + 87$\\
&&$b = r_{1}\cdot q_{2} + r_{2}$ &$102 = 87\cdot 1 + 15$\\
&&$r_{1} = r_{2}\cdot q_{3} + r_{3}$ &$87 = 15\cdot 5 + 12$\\
&&$r_{2} = r_{3}\cdot q_{4} + r_{4}$ &$15 = 12\cdot 1 + \boxed{\mathbf{3}}$\\
&&$r_{3} = r_{4}\cdot q_{5} + r_{5}$ &$12 = 3\cdot 4 + 0$\\
$1529$ &$170$ &$a = b\cdot q_{1} + r_{1}$ &$1529 = 170\cdot 8 + 169$\\
&&$b = r_{1}\cdot q_{2} + r_{2}$ &$170 = 169\cdot 1 + \boxed{\mathbf{1}}$\\
&&$r_{1} = r_{2}\cdot q_{3} + r_{3}$ &$169 = 1\cdot 169 + 0$\\
\hline
$13758$ &$21$ &$a = b\cdot q_{1} + r_{1}$ &$13758 = 21\cdot 655 + \boxed{\mathbf{3}}$\\
&&$b = r_{1}\cdot q_{2} + r_{2}$ &$21 = 3\cdot 7 + 0$\\
\hline
$210$ &$45$ &$a = b\cdot q_{1} + r_{1}$ &$210 = 45\cdot 4 + 30$\\
&&$b = r_{1}\cdot q_{2} + r_{2}$ &$45 = 30\cdot 1 + \boxed{\mathbf{15}}$\\
&&$r_{1} = r_{2}\cdot q_{3} + r_{3}$ &$30 = 15\cdot 2 + 0$\\
\hline
$1209$ &$102$ &$a = b\cdot q_{1} + r_{1}$ &$1209 = 102\cdot 11 + 87$\\
&&$b = r_{1}\cdot q_{2} + r_{2}$ &$102 = 87\cdot 1 + 15$\\
&&$r_{1} = r_{2}\cdot q_{3} + r_{3}$ &$87 = 15\cdot 5 + 12$\\
&&$r_{2} = r_{3}\cdot q_{4} + r_{4}$ &$15 = 12\cdot 1 + \boxed{\mathbf{3}}$\\
&&$r_{3} = r_{4}\cdot q_{5} + r_{5}$ &$12 = 3\cdot 4 + 0$\\
\hline
\hline
\end{longtable}
@ -7559,18 +7557,18 @@ Wir verwenden die Berechnungen aus der Tabelle in SKA \ref{ska:5:ex:6}.
$a$ &$b$ &Rest (symbolisch) &Rest (Werte)\\
\hline
\endhead
$1529$ &$170$ &$r_{1} = a - 8\cdot b$ &$169 = 1\cdot a + -8\cdot b$\\
&&$r_{2} = b - 1\cdot r_{1}$ &$\boxed{1 = \mathbf{-1}\cdot a + \mathbf{9}\cdot b}$\\
\hline
$13758$ &$21$ &$r_{1} = a - 655\cdot b$ &$\boxed{3 = \mathbf{1}\cdot a + \mathbf{-655}\cdot b}$\\
\hline
$210$ &$45$ &$r_{1} = a - 4\cdot b$ &$30 = 1\cdot a + -4\cdot b$\\
&&$r_{2} = b - 1\cdot r_{1}$ &$\boxed{15 = \mathbf{-1}\cdot a + \mathbf{5}\cdot b}$\\
\hline
$1209$ &$102$ &$r_{1} = a - 11\cdot b$ &$87 = 1\cdot a + -11\cdot b$\\
&&$r_{2} = b - 1\cdot r_{1}$ &$15 = -1\cdot a + 12\cdot b$\\
&&$r_{3} = r_{1} - 5\cdot r_{2}$ &$12 = 6\cdot a + -71\cdot b$\\
&&$r_{4} = r_{2} - 1\cdot r_{3}$ &$\boxed{3 = \mathbf{-7}\cdot a + \mathbf{83}\cdot b}$\\
$1529$ &$170$ &$r_{1} = a - 8\cdot b$ &$169 = 1\cdot a + -8\cdot b$\\
&&$r_{2} = b - 1\cdot r_{1}$ &$\boxed{1 = \mathbf{-1}\cdot a + \mathbf{9}\cdot b}$\\
\hline
$13758$ &$21$ &$r_{1} = a - 655\cdot b$ &$\boxed{3 = \mathbf{1}\cdot a + \mathbf{-655}\cdot b}$\\
\hline
$210$ &$45$ &$r_{1} = a - 4\cdot b$ &$30 = 1\cdot a + -4\cdot b$\\
&&$r_{2} = b - 1\cdot r_{1}$ &$\boxed{15 = \mathbf{-1}\cdot a + \mathbf{5}\cdot b}$\\
\hline
$1209$ &$102$ &$r_{1} = a - 11\cdot b$ &$87 = 1\cdot a + -11\cdot b$\\
&&$r_{2} = b - 1\cdot r_{1}$ &$15 = -1\cdot a + 12\cdot b$\\
&&$r_{3} = r_{1} - 5\cdot r_{2}$ &$12 = 6\cdot a + -71\cdot b$\\
&&$r_{4} = r_{2} - 1\cdot r_{3}$ &$\boxed{3 = \mathbf{-7}\cdot a + \mathbf{83}\cdot b}$\\
\hline
\hline
\end{longtable}
@ -8237,10 +8235,10 @@ Für $x=[2]$ und $y=[3]$ gilt $x,y\neq [0]$ und aber $xy=[2\cdot 3]=[6]=[0]$.
Restberechnung (symbolisch) &Restberechnung (Werte)\\
\hline
\endhead
$a = b\cdot q_{1} + r_{1}$ &$103 = 21\cdot 4 + 19$\\
$b = r_{1}\cdot q_{2} + r_{2}$ &$21 = 19\cdot 1 + 2$\\
$r_{1} = r_{2}\cdot q_{3} + r_{3}$ &$19 = 2\cdot 9 + \boxed{\mathbf{1}}$\\
$r_{2} = r_{3}\cdot q_{4} + r_{4}$ &$2 = 1\cdot 2 + 0$\\
$a = b\cdot q_{1} + r_{1}$ &$103 = 21\cdot 4 + 19$\\
$b = r_{1}\cdot q_{2} + r_{2}$ &$21 = 19\cdot 1 + 2$\\
$r_{1} = r_{2}\cdot q_{3} + r_{3}$ &$19 = 2\cdot 9 + \boxed{\mathbf{1}}$\\
$r_{2} = r_{3}\cdot q_{4} + r_{4}$ &$2 = 1\cdot 2 + 0$\\
\hline
\hline
\end{longtable}
@ -8254,9 +8252,9 @@ Für $x=[2]$ und $y=[3]$ gilt $x,y\neq [0]$ und aber $xy=[2\cdot 3]=[6]=[0]$.
Rest (symbolisch) &Rest (Werte)\\
\hline
\endhead
$r_{1} = a - 4\cdot b$ &$19 = 1\cdot a + -4\cdot b$\\
$r_{2} = b - 1\cdot r_{1}$ &$2 = -1\cdot a + 5\cdot b$\\
$r_{3} = r_{1} - 9\cdot r_{2}$ &$\boxed{1 = \mathbf{10}\cdot a + \mathbf{-49}\cdot b}$\\
$r_{1} = a - 4\cdot b$ &$19 = 1\cdot a + -4\cdot b$\\
$r_{2} = b - 1\cdot r_{1}$ &$2 = -1\cdot a + 5\cdot b$\\
$r_{3} = r_{1} - 9\cdot r_{2}$ &$\boxed{1 = \mathbf{10}\cdot a + \mathbf{-49}\cdot b}$\\
\hline
\hline
\end{longtable}
@ -9054,7 +9052,7 @@ für alle linearen Unterräume, $U\subseteq V$.
\textbf{zu zeigen}, dass ein $x\in U$ existiert mit
\begin{mathe}[mc]{rcl}
\eqtag[eq:0:quiz:9]{$\ast$}
\eqtag[eq:0:quiz:9]
(\psi\circ\phi)(x) &= &z.\\
\end{mathe}
@ -9102,15 +9100,18 @@ für alle linearen Unterräume, $U\subseteq V$.
\end{einzug}
\begin{rem*}
Wir können in der Tat zeigen, die umgekehrte Richtung auch gilt:
Wir können in der Tat zeigen, dass die umgekehrte Richtung auch gilt:
Angenommen, $\psi\circ\phi$ sei surjektiv.
Dann gilt
$W\supseteq\psi(V)\supseteq\psi(\phi(U))=(\psi\circ\phi)(U)=W$,
und somit $\psi(V)=W$,
sodass \eqcref{it:1:quiz:9} gilt.
Und für alle $y\in V$, wegen Surjektivität von $\psi\circ\phi$,
existiert ein $x\in U$, so dass $\psi(y)=(\psi\circ\phi)(x)$.
Daraus folgt
Für \eqcref{it:2:quiz:9} brauchen wir nur die $\supseteq$-Inklusion zu zeigen,
da die $\subseteq$-Inklusion offensichtlich wahr ist.
Sei also $y\in V$ beliebig.
Wegen Surjektivität von $\psi\circ\phi$ existiert nun ein $x\in U$,
so dass $\psi(y)=(\psi\circ\phi)(x)$.
Beobachte man, dass
\begin{mathe}[mc]{rcccl}
\psi(y-\phi(x))
@ -9127,8 +9128,10 @@ für alle linearen Unterräume, $U\subseteq V$.
&\in &\ker(\psi)+\range(\phi).\\
\end{mathe}
Also gilt die $\supseteq$-Inklusion in \eqcref{it:2:quiz:9}.
Und offensichtlich gilt die $\subseteq$-Inklusion in \eqcref{it:1:quiz:9}.
Damit haben wir bewiesen, dass $V\subseteq \ker(\psi)+\range(\phi)$
(d.\,h. die $\supseteq$-Inklusion in \eqcref{it:2:quiz:9}).\\
Darum gilt:
$\psi\circ\phi$ surjektiv $\Rightarrow$ \eqcref{it:1:quiz:9}+\eqcref{it:2:quiz:9} gelten.
\end{rem*}
%% ********************************************************************************